Đến nội dung

Hình ảnh

Inequalities From 2016 Mathematical Olympiads

* * * * * 3 Bình chọn

  • Please log in to reply
Chủ đề này có 139 trả lời

#121
superpower

superpower

    Sĩ quan

  • Thành viên
  • 492 Bài viết

Bài 54 (Azerbaijan JBMO TST). Với $a,b,c$ là ba số thực dương thỏa mãn điều kiện $a+b+c=3.$ Chứng minh rằng
\[2(ab+ac+bc)-3abc \geqslant a \sqrt{\frac{b^2+c^2}{2}}+b \sqrt{\frac{c^2+a^2}{2}}+c \sqrt{\frac{a^2+b^2}{2}}.\]

Em xin giải bài 54 bằng SOS

BĐT $<=> \frac{(ab+bc+ca)(a+b+c) -9abc}{3} \geq \sum a( \sqrt{\frac{b^2+c^2}{2}} - \frac{b+c}{2} )$

Nhân liệp hiệp và phân tích bình phương SOS, ta được

$\frac{\sum c(a-b)^2 }{3} \geq \sum \frac{c(a-b)^2}{\sqrt{8(a^2+b^2)} +2(a+b) }  $ 

$<=> \sum (\frac{c}{3} - \frac{c}{\sqrt{8(a^2+b^2)} +2(a+b) }) (a-b)^2 \geq 0 $

Mà ta có $2(a^2+b^2 ) \geq (a+b)^2 $

Do đó $\sqrt{8(a^2+b^2)} +2(a+b)  \geq 4(a+b) $

Ta chứng minh  1 bđt yếu hơn 

$\sum c. \frac{4(a+b) -3}{12(a+b) } (a-b)^2 \geq 0 $

Đặt $S_c = \frac{4(a+b) -3}{12(a+b) } c  $

Tương tự cho $S_a , S_b $ 

Đưa bđt về dạng $\sum S_a(b-c)^2 \geq 0 $ thì ta có

$S_c \geq 0 <=> 4(a+b) \geq 3 $ đúng do $a \geq 1 $

$S_b \geq 0 <=> 4(a+c) \geq 3 $ đúng do $a \geq 1 $

Bây giờ ta chứng minh $b^2S_a + a^2S_b \geq 0 <=> 4\frac{a+b}{3} \geq \frac{a}{a+c} +\frac{b}{b+c} $

Mà dễ thấy $\frac{a}{a+c} + \frac{b}{b+c} \leq 2 $

Do đó , ta cần chứng minh $4(a+b) \geq 6 $ mà này thì hiển nhiên do $3 = a+b+c \leq 2(a+b) => 4(a+b) \geq 6 $  

Vậy, ta có điều phải chứng minh



#122
Nguyenhuyen_AG

Nguyenhuyen_AG

    Trung úy

  • Thành viên nổi bật 2016
  • 945 Bài viết

Em thì post đề ủng hộ chứ không biết giải, nếu đề có trùng với các đề trước thì mong anh Huyện và mọi người thông cảm !

Bài 55 (Proposed for 1999 USAMO): Cho $x,y,z>1$.Chứng minh rằng

$x^{x^{2}+2yz}y^{y^{2}+2zx}z^{z^{2}+2xy}\geq (xyz)^{xy+yz+zx}.$

 

Bài 56 (APMO,2005): Cho $a,b,c$ là các số thực dương thỏa mãn điều kiện $abc=8$.Chứng minh rằng

$\frac{a^2}{\sqrt{(a^3+1)(b^3+1)}}+\frac{b^2}{\sqrt{(b^3+1)(c^3+1)}}+\frac{c^2}{\sqrt{(c^3+1)(a^3+1)}}\geq \frac{4}{3}.$

 

Topic này mình chỉ đăng bất đẳng thức ở các kỳ thi vào năm 2016.


Nguyen Van Huyen
Ho Chi Minh City University Of Transport

#123
Nguyenhuyen_AG

Nguyenhuyen_AG

    Trung úy

  • Thành viên nổi bật 2016
  • 945 Bài viết

Bài 55 (2016 CWMO). Cho $a_1,a_2,\ldots,a_n$ là các số thực không âm và đặt $\displaystyle S_k= \sum\limits_{i=1}^{k}a_i,\;(1\le k\le n).$  Chứng minh rằng

$$\sum\limits_{i=1}^{n}\left(a_i \cdot S_i\sum\limits_{j=i}^{n}a^2_j\right)\le \sum\limits_{i=1}^{n}\left(a_iS_i\right)^2.$$

Bài 56 (MEMO). Cho số nguyên dương $n \ge 2$ và $x_1, x_2, \ldots, x_n$ là các số thực thỏa mãn đồng thời

  • $x_j > -1$ với mọi $j = 1, 2,  \ldots, n.$
  • $x_1 + x_2 + \cdots + x_n = n.$

Chứng minh rằng

$$ \sum_{j = 1}^{n} \frac{1}{1 + x_j} \ge \sum_{j = 1}^{n} \frac{x_j}{1 + x_j^2}.$$
Đẳng thức xảy ra khi vào ?

 

Bài 57 (Iran MO 3rd Round Finals). Cho ba số thực dương $a,b,c$ thỏa mãn điều kiện $abc=1.$ Chứng minh rằng
$$\frac {a+b}{(a+b+1)^2}+\frac {b+c}{(b+c+1)^2}+\frac {c+a}{(c+a+1)^2} \geqslant \frac {2}{a+b+c}.$$



 


Bài viết đã được chỉnh sửa nội dung bởi Nguyenhuyen_AG: 02-09-2016 - 08:43

Nguyen Van Huyen
Ho Chi Minh City University Of Transport

#124
Nguyenhuyen_AG

Nguyenhuyen_AG

    Trung úy

  • Thành viên nổi bật 2016
  • 945 Bài viết

Bài 57 (KMO Winter School). Cho $n$ số thực dương $x_1,x_2,\ldots,x_n.$ Chứng minh rằng
\[(1+x_1)(1+x_1+x_2)\cdots(1+x_1+x_2+\cdots+x_n)\geq\sqrt{(n+1)^{n+1} x_1x_2\cdots x_n}.\]
P/s. Bài này khá hay nhưng không mới, là một đề thi cũ của Nga. :)


Nguyen Van Huyen
Ho Chi Minh City University Of Transport

#125
tpdtthltvp

tpdtthltvp

    Trung úy

  • Điều hành viên THCS
  • 831 Bài viết

Bài 57 (Iran MO 3rd Round Finals). Cho ba số thực dương $a,b,c$ thỏa mãn điều kiện $abc=1.$ Chứng minh rằng

$$\frac {a+b}{(a+b+1)^2}+\frac {b+c}{(b+c+1)^2}+\frac {c+a}{(c+a+1)^2} \geqslant \frac {2}{a+b+c}.$$

 

Áp dụng BĐT $Cauchy-schwarz,$ ta có:

$$(a+b+c)(a+b+\frac{1}{c})\geq (a+b+1)^2$$

Kết hợp với $abc=1$

$$\sum \frac{a+b}{(a+b+1)^2}\geq \sum \frac{a+b}{(a+b+c)(a+b+ab)}$$

Cuối cùng ta chỉ cần chứng minh:

$$\sum \frac{a+b}{a+b+ab}\geq 2\Leftrightarrow \sum \frac{ab}{a+b+ab}\leq 1$$

Đặt $a=\frac{1}{x^3};b=\frac{1}{y^3},c=\frac{1}{z^3}$ thì $xyz=1$ và:

$$\sum \frac{ab}{a+b+ab}=\sum \frac{1}{\frac{1}{a}+\frac{1}{b}+1}=\sum \frac{1}{x^3+y^3+xyz}\leq \sum \frac{1}{xy(x+y)+xyz}=\sum \frac{z}{xyz(x+y+z)}=\sum \frac{z}{x+y+z}=1.$$

Do đó ta có đpcm.


Bài viết đã được chỉnh sửa nội dung bởi tpdtthltvp: 05-09-2016 - 12:18

$\color{red}{\mathrm{\text{How I wish I could recollect, of circle roud}}}$

$\color{red}{\mathrm{\text{The exact relation Archimede unwound ! }}}$

 


#126
Nguyenhuyen_AG

Nguyenhuyen_AG

    Trung úy

  • Thành viên nổi bật 2016
  • 945 Bài viết

Bài 59 (China Second Round). Cho $ a,b,c$ là ba số thực thỏa mãn điều kiện $a>b^2$ và $b>c^2 .$ Tìm giá trị lớn nhất của biểu thức $$(a-b^2)(b-c^2)(c-a^2).$$
 


Nguyen Van Huyen
Ho Chi Minh City University Of Transport

#127
Kalari499

Kalari499

    Binh nhất

  • Thành viên mới
  • 37 Bài viết

Bài trên cứ cảm giác như thiếu điều kiện gì đó ấy @@



#128
MATHVNkakaka

MATHVNkakaka

    Lính mới

  • Thành viên mới
  • 8 Bài viết

Đúng rồi em, mình giải ở đây luôn. :)

dạ anh ơi cho em hỏi là dấu sigma được dùng như thế nào ? có phải là tổng không ?



#129
Nguyenhuyen_AG

Nguyenhuyen_AG

    Trung úy

  • Thành viên nổi bật 2016
  • 945 Bài viết

dạ anh ơi cho em hỏi là dấu sigma được dùng như thế nào ? có phải là tổng không ?

 

Ký hiệu $\sum $ là tổng.


Nguyen Van Huyen
Ho Chi Minh City University Of Transport

#130
MATHVNkakaka

MATHVNkakaka

    Lính mới

  • Thành viên mới
  • 8 Bài viết

Ký hiệu $\sum $ là tổng

có thể nói rõ cho em là tại sao em làm đúng theo diễn đàn mà k gõ được kí hiêu toán học ko ?



#131
Mr Cooper

Mr Cooper

    Sĩ quan

  • Thành viên
  • 496 Bài viết

Bài 17 (Azerbaijan Junior Mathematical Olympiad). Với $x,\,y,\,z$ là ba số thực khác $0.$ Chứng minh rằng $$\sqrt {x^2+\frac {1}{y^2}}+ \sqrt {y^2+\frac {1}{z^2}}+ \sqrt {z^2+\frac {1}{x^2}}\geq 3\sqrt {2}. $$

 

 

Bài 17:

Áp dụng bất đẳng thức Minkowxki ta có:

\[\sqrt {{x^2} + \frac{1}{{{y^2}}}}  + \sqrt {{y^2} + \frac{1}{{{z^2}}}}  + \sqrt {{z^2} + \frac{1}{{{x^2}}}}  \ge \sqrt {{{\left( {x + y + z} \right)}^2} + {{\left( {\frac{1}{x} + \frac{1}{y} + \frac{1}{z}} \right)}^2}} \]

Áp dụng bất đẳng thức AM - GM ta có:

\[\sqrt {{{\left( {x + y + z} \right)}^2} + {{\left( {\frac{1}{x} + \frac{1}{y} + \frac{1}{z}} \right)}^2}}  \ge \sqrt {9\sqrt[3]{{{x^2}{y^2}{z^2}}} + 9\frac{1}{{\sqrt[3]{{{x^2}{y^2}{z^2}}}}}}  \ge 3\sqrt 2 \]

Dấu đẳng thức xảy ra khi và chỉ khi: x = y = z = 1



#132
Nguyenphuctang

Nguyenphuctang

    Sĩ quan

  • Banned
  • 499 Bài viết

Ký hiệu $\sum $ là tổng.

Kí hiệu $\sum $ được chia làm 2 loại:
+Sigma cyclic : Tổng hoá vị.
Kí hiệu: $\sum_{cyc}^{}$
+Sigma symetric : Tổng đối xứng 
Kí hiệu: $\sum_{sym}^{}$ 
Ta có thể ghi  $\sum $ nếu không sợ bị nhầm nhẫm 


Bài viết đã được chỉnh sửa nội dung bởi Nguyenphuctang: 07-11-2016 - 17:58


#133
nhatkinan

nhatkinan

    Hạ sĩ

  • Thành viên
  • 56 Bài viết

Không biết giải bài ở đây có vi phạm không anh nhỉ :( 
$VT=\sum \frac{a^3+1+1+6}{a^3(b+c)} \ge \sum \frac{3a+6}{a^3(b+c)}=\sum \frac{3(a+2)}{a^3(b+c)}$ 
Ta sẽ chứng minh $\sum \frac{3(a+2)}{a^3(b+c)} \ge \frac{27}{2}$ (*)
Chợt nhận thấy bài toán quen thuộc của IMO 1995  
Nếu $abc=1$ thì $\sum \frac{1}{a^3(b+c)} \ge \frac{3}{2}$ 
Áp dụng suy ra $\frac{6}{a^3(b+c)} \ge 9$ 
Lại có $\sum \frac{3}{a^2(b+c)}=\sum \frac{3(bc)^2}{b+c} \ge \frac{3(\sum ab)^2}{2\sum a}$ (1) 
Lại có $(\sum ab)^2 \ge 3.abc(a+b+c)$ nên từ (1) suy ra $\frac{3(\sum ab)^2}{2\sum a} \ge \frac{9}{2}$ 
Cộng lại suy ra (*) được chứng minh 
Vậy giá trị nhỏ nhất là $\frac{27}{2}$ khi và chỉ khi $a=b=c=1$

bài này nếu mh nhớ kh nhầm thì từng đăng lên báo toán hk tuổi trẻ rồi



#134
dungxibo123

dungxibo123

    Sĩ quan

  • Thành viên
  • 330 Bài viết

Từ giả thiết, áp dụng bđt AM-GM, ta được:
$1\leq \sqrt{abc}(\sqrt{a}+\sqrt{b}+\sqrt{c})\leq \frac{1}{9}(\sqrt{a}+\sqrt{b}+\sqrt{c})(a+b+c)(\sqrt{a}+\sqrt{b}+\sqrt{c})\leq \frac{1}{3}(a+b+c)^2\Rightarrow a+b+c\geq \sqrt{3}$
Đẳng thức xảy ra khi a=b=c=$\frac{1}{\sqrt{3}}$

1\leq\sum a\sqrt{bc}\leq\sum ab+bc+ca\leq\frac{(a+b+c)^{2}}{3}

$\Rightarrow a+b+c \geq \sqrt{3}$

em nghĩ cách này khá tự nhiên và khái quát :)) 
P/s: hồi nào ra sách vậy em chơ mãi mà chưa thấy


myfb : www.facebook.com/votiendung.0805
~~~~~~~~~~~~~~~~~~~~~~~~~~~~~~~~~~~~~o0o~~~~~~~~~~~~~~~~~~~~~~~~~~~~~~~~~~~~~~
SỢ HÃI giúp ta tồn tại

NGHỊ LỰC giúp ta đứng vững

KHÁT VỌNG giúp ta tiến về phía trước

Võ Tiến Dũng  

:like  :like  :like  :like  :like 

 

 


#135
dungxibo123

dungxibo123

    Sĩ quan

  • Thành viên
  • 330 Bài viết

Bài 60 : ( Polish MO 2009) 

cho $n \geq 1$ và $ a,b,c>0$

chứng minh $\sum \frac{a^{n+1}}{b+c}\geq(\sum \frac{a^{n}}{b+c})\sqrt[n]{\frac{\sum a^{n}}{3}}$

 

 

em xin ké bài này :)) đăng trên diễn đàn mãi mà chưa thấy mem nào trả lời nên :)) cho em ké ạ


myfb : www.facebook.com/votiendung.0805
~~~~~~~~~~~~~~~~~~~~~~~~~~~~~~~~~~~~~o0o~~~~~~~~~~~~~~~~~~~~~~~~~~~~~~~~~~~~~~
SỢ HÃI giúp ta tồn tại

NGHỊ LỰC giúp ta đứng vững

KHÁT VỌNG giúp ta tiến về phía trước

Võ Tiến Dũng  

:like  :like  :like  :like  :like 

 

 


#136
DNThi

DNThi

    Lính mới

  • Thành viên mới
  • 9 Bài viết

Bài 17:

Áp dụng bất đẳng thức Minkowxki ta có:

\[\sqrt {{x^2} + \frac{1}{{{y^2}}}}  + \sqrt {{y^2} + \frac{1}{{{z^2}}}}  + \sqrt {{z^2} + \frac{1}{{{x^2}}}}  \ge \sqrt {{{\left( {x + y + z} \right)}^2} + {{\left( {\frac{1}{x} + \frac{1}{y} + \frac{1}{z}} \right)}^2}} \]

Áp dụng bất đẳng thức AM - GM ta có:

\[\sqrt {{{\left( {x + y + z} \right)}^2} + {{\left( {\frac{1}{x} + \frac{1}{y} + \frac{1}{z}} \right)}^2}}  \ge \sqrt {9\sqrt[3]{{{x^2}{y^2}{z^2}}} + 9\frac{1}{{\sqrt[3]{{{x^2}{y^2}{z^2}}}}}}  \ge 3\sqrt 2 \]

Dấu đẳng thức xảy ra khi và chỉ khi: x = y = z = 1

AM-GM hai lần

\[\sqrt {{x^2} + \frac{1}{{{y^2}}}}  + \sqrt {{y^2} + \frac{1}{{{z^2}}}}  + \sqrt {{z^2} + \frac{1}{{{x^2}}}}  \geq \sqrt{2} \left(\sqrt{\frac{x}{y}}+\sqrt{\frac{y}{z}}+\sqrt{\frac{z}{x}}\right) \geq 3\sqrt{2}\]


Bài viết đã được chỉnh sửa nội dung bởi DNThi: 22-03-2017 - 15:15

Tuyển Tập Đề Thi Tuyển Sinh & Học Sinh Giỏi Toán

http://molympiad.net/


#137
DNThi

DNThi

    Lính mới

  • Thành viên mới
  • 9 Bài viết

Bài 55 (2016 CWMO). Cho $a_1,a_2,\ldots,a_n$ là các số thực không âm và đặt $\displaystyle S_k= \sum\limits_{i=1}^{k}a_i,\;(1\le k\le n).$  Chứng minh rằng

$$\sum\limits_{i=1}^{n}\left(a_i \cdot S_i\sum\limits_{j=i}^{n}a^2_j\right)\le \sum\limits_{i=1}^{n}\left(a_iS_i\right)^2.$$

 

Ta có

 

 

$$VT=\sum_{i=1}^{n}\left(a_{i}\left(\sum_{k=1}^{i}a_{i}\right)\left(\sum_{j=i}^{n}a_{j}^{2}\right)\right)=\sum_{i=1}^{n}\left(a_{i}\sum_{1\leq k\leq i\leq j}^{n}a_{k}a_{j}^{2}\right)=\sum_{1\leq k\leq i\leq j\leq n}^{n}a_{k}a_{i}a_{j}^{2}=\sum_{1\leq i\leq j\leq n}^{n}a_{i}^{2}a_{j}^{2}+\sum_{1\leq k<i\leq j\leq n}^{n}a_{k}a_{i}a_{j}^{2}$$

 

 

 

do đó bđt cần chứng minh tương đương với

 

$$0\leq\sum_{1\leq j<k\leq i\leq n}a_{j}a_{k}a_{i}^{2}$$

 

Đẳng thức xảy ra khi $a_i=0$


Bài viết đã được chỉnh sửa nội dung bởi DNThi: 23-03-2017 - 00:12

Tuyển Tập Đề Thi Tuyển Sinh & Học Sinh Giỏi Toán

http://molympiad.net/


#138
DNThi

DNThi

    Lính mới

  • Thành viên mới
  • 9 Bài viết

Bản gõ $\LaTeX$ tổng hợp một số đề và lời giải chưa hoàn chỉnh, file ineq-2016-mo.lyx được phép tải về đề một bạn nào đó tiếp tục cho hoàn thành.

 

 

File gửi kèm


Tuyển Tập Đề Thi Tuyển Sinh & Học Sinh Giỏi Toán

http://molympiad.net/


#139
aodaidanang

aodaidanang

    Lính mới

  • Thành viên mới
  • 2 Bài viết

Hình như bài 59 thiếu điều kiện gì ấy :(


Áo dài Đà Nẵng https://goo.gl/SNZ9Kq

Thuê áo dài Đà Nẵng https://goo.gl/SNZ9Kq

Áo dài Đà Nẵng

Thuê áo dài Đà Nẵng


#140
toanhoc2017

toanhoc2017

    Thiếu úy

  • Banned
  • 628 Bài viết
co bai tap dua len anh em tham khao nhe




2 người đang xem chủ đề

0 thành viên, 2 khách, 0 thành viên ẩn danh